You are on page 1of 36

Practice Questions 1

B1 C8 2
1. A physical therapy plan of care for a child with spastic cerebral palsy who is three years-old chronologically and
cognitively, but at a six month-old gross developmental level would include:
A. reaching for a black and white object while in the supine position.
B. reaching for a multicolored object while in an unsupported standing position.
C. reaching for a multicolored object while in an unsupported, guarded sitting position.
D. visually tracking a black and white object held nine inches from his face.

B2 C1 2
2. A patient is receiving grade III mobilizations to regain normal midthoracic extension. After three sessions he
complains of localized pain that persists for greater than 24 hours. Your treatment should:
A. change to grade II mobilizations to reduce the joint and soft tissue irritation.
B. change to grade IV mobilizations to stretch through the pain.
C. change to self stretching activities because the patient does not tolerate mobilization.
D. continue with grade III mobilizations followed by a cold pack to the thoracic spine.

B3 C7 3
3. A therapist wishes to use behavior modification techniques as part of her treatment to help shape the behavioral
responses of a 22 year-old patient recovering from traumatic brain injury. The BEST choice is to:
A. allow the patient enough time for self correction of the behavior.
B. encourage the staff to tell the patient which behaviors are correct and which are not.
C. reprimand the patient every time an undesirable behavior occurs.
D. use frequent reinforcements for all desired behaviors.

B4 C2 3
4. Which is NOT a useful intervention for a patient with multiple sclerosis who presents with a primary deficit of
dysmetria?
A. isokinetic training using low resistance at fast movement speeds.
B. PNF patterns using carefully graded resistance and slow reversals.
C. pool therapy using moderate water temperatures.
D. weight cuffs to distal extremities during functional training.

B5 C6 3
5. A 16 year old patient suffered fractures of C4 and C5 following trauma received in a motor vehicle accident.
Maximum stabilization of his cervical spine can BEST be achieved with:
A. four-poster orthosis.
B. halo orthosis.
C. Milwaukee orthosis.
D. soft collar.

B6 C3 2
6. A patient had been on oxygen, but it was discontinued by physician order yesterday. During physical therapy, the
patient becomes short of breath and requests supplemental oxygen. The patients SaO2 is measured at 90%. The
decision that is NOT appropriate is:
A. administer supplemental O2.
B. allow the patient to rest.
C. continue to monitor SaO2.
D. encourage an efficient breathing pattern.

B7 C2 1
7. A 26 year-old presents with weakness and atrophy of the biceps brachii resulting from an open fracture of the
humerus. Your examination includes needle electromyography of the biceps. The muscle response you would
anticipate after the needle was inserted and prior to asking the patient to contract the muscle is:
A. electrical silence.
B. fibrillation potentials.
C. interference patterns.
D. polyphasic potentials.

373406391.doc
2 Practice Questions

B8 C4 2
8. You are a new staff member on the oncology unit of a large medical center. Your first referral is for strengthening
and ambulation for a 42 year-old women with ovarian cancer. She is undergoing radiation therapy following a
surgical hysterectomy. Her current platelet count is 17,000. The MOST appropriate treatment activity for this
patient at this time is:
A. passive ROM exercise.
B. progressive stair climbing using weighted belts.
C. resistance training at 30% one repetition max.
D. resistance training at 50 % one repetition max.

B9 C1 2
9. A forty-three year-old male office worker who is a long term smoker now with emphysema has been referred to
physical therapy. He complains of increased pain and tingling in both hands after sitting at his desk for longer than
one hour. The MOST effective physical therapy intervention is:
A. mobilization of the first rib and stretching of middle trapezius muscle due to long term postural changes.
B. strengthening of the upper trapezius and pectoralis major muscles for early thoracic outlet syndrome
symptoms.
C. stretching of scalenes and sternocleidomastoid muscles for early thoracic outlet syndrome symptoms.
D. stretching of the pectoralis major and rhomboid muscles due to long term postural changes.

B10 C8 2
10. A 68 year-old patient with diabetes is exercising in your P.T. clinic. He reports feeling weak, dizzy, and somewhat
nauseous. You notice that he is sweating profusely and is unsteady on his feet. He then becomes confused about
where he is. You determine his problems are most likely due to and your immediate course of action should be:
A. an insulin reaction; you call for emergency services.
B. fatigue; you insist that he rest until his symptoms resolve.
C. hyperglycemia; you have a nurse administer an insulin injection.
D. hypoglycemia; you immediately administer orange juice and notify his physician.

B11 C2 2
11. A patient with postpolio syndrome has started attending an aerobic exercise program as an outpatient. When she
does not show up for her scheduled session, you telephone to find out what is the matter. She tells you that she is
very tired and has not been able to get out of bed for the last two days. Her muscles ache and she is afraid to
continue with the exercise class. You decide to:
A. discharge her from the program since exercise is counterproductive in postpolio syndrome.
B. modify her exercise prescription to decrease the frequency to once a week for an hour session, keeping the
intensity moderate.
C. modify her exercise prescription to decrease the intensity and duration but maintain a frequency of 3
times/week.
D. reschedule her workouts for early morning when she feels refreshed.

B12 C7 3
12. You are the primary clinical instructor for a final affiliation physical therapy student. As you work with this student
you become aware that she tends to process information all at once, not in an ordered step-by-step manner. Your
best strategy to ensure adequate learning for this student is to:
A. focus the student on learning important relationships and concepts.
B. focus the student on objective information and interrelationships.
C. provide real-life examples that link learning to her personal experiences.
D. redirect her to process information in a step-by-step manner.

B13 C7 2
13. A 69 year-old male falls and breaks his hip. He undergoes a total hip replacement at the local medical center. The
physician orders physical therapy two days after surgery. The patient is covered by Medicare and is still a patient
in an acute care facility. The appropriate frequency of treatment would be:
A. as often as possible, for the first two weeks based on the therapists caseload.
B. daily, following the initial examination.
C. every other day.
D. three to four times per day to facilitate early discharge.

373406391.doc
Practice Questions 3

B14 C3 1
14. A patient is taking a drug from the sympathomimetic group, Albuterol. Which is NOT an expected effect of this
drug?
A. bronchodilation.
B. exercise-induced bronchospasm.
C. hypertension
D. tachycardia.

B15 C5 2
15. A 48 year-old female patient is recovering from a total knee replacement. Following her surgery, you receive a
referral to teach therapeutic exercises and a home program. The intervention not appropriate during the early
postoperative phase is:
A. ankle pumps.
B. isotonic exercises using ankle weights.
C. seated knee extension.
D. straight leg raising.

B16 C6 2
16. You determine that a patient is walking with a backward trunk lean as he takes full weight on his right leg. He also
demonstrates great difficulty going up ramps. The BEST intervention to remediate his problem is to:
A. strengthen hip extensors through bridging.
B. strengthen knee extensors with weights, using 80% one repetition maximum.
C. stretch hip abductors through sidelying positioning.
D. stretch hip flexors through prone lying.

B17 C4 2
17. If a patient has developed a thick eschar secondary to a full thickness burn, the antibacterial agent MOST effective
for infection control is:
A. silver nitrate.
B. Sulfamylon.
C. Travase.
D. nitrofurazone.

B18 C8 1
18. An 89 year-old has gradually lost much of his functional vision over the last 4 years. He complains his vision is
foggy and he has difficulty reading. He mistakes images directly in front of him, especially in bright light. He
does better walking across a room and is able to locate items in his environment with his peripheral vision when
items are located to both sides. Based on his descriptions, you suspect he may be presenting with:
A. cataracts.
B. glaucoma.
C. homonymous hemianopsia.
D. tunnel vision secondary to overdoses of steroid medications.

B19 C2 2
19. Following a traumatic brain injury, a 26 year-old woman is inconsistently oriented to time and place. She is unable
to remember recent events and shows little or no carry-over for new learning. The PRIMARY goal of
rehabilitation at this stage of recovery is to promote:
A. an environmental and daily structure in which the patient is best able to process stimuli cognitively.
B. functional independence in bed mobility and transfers.
C. increased arousal and attention through the use of sensory stimulation techniques.
D. independence in problem solving skills in variable environments.

373406391.doc
4 Practice Questions

B20 C2 2
20. A 65 year-old woman suffered a right CVA one month ago. She demonstrates moderate tone in her left upper
extremity (predominantly flexor tone). Her major problem at this time is a lack of voluntary movement control and
her left upper extremity shows minimal active movement with 1/4 inch subluxation of the shoulder. An appropriate
initial activity would be:
A. PNF D2 flexion pattern, left upper extremity.
B. quadruped, rocking from side to side.
C. sitting, left active shoulder protraction with extended arm.
D. sitting, weight bearing on extended left arm, weight shifting.

B21 C7 3
21. A therapist conducts a study of the effectiveness of hot and cold in treating patients with pain. He recruits 200
patients for each treatment group. The pain instrument he uses has a possible total score of 50, with 50 being the
worst pain. His data analysis reveals that Group A (heat modalities) has a mean score of 33 with a standard
deviation of 1.0 while Group B (cold modalities) had a mean of 35 with a standard deviation of 6.0. Based on this
data the conclusion one should reach is:
A. cold has a greater effect on pain relief than heat.
B. heat has a greater effect on pain relief than cold.
C. the spread of scores with cold treatment demonstrates variability is greater.
D. the spread of scores with heat treatment demonstrates variability is greater.

B22 C1 1
22. The therapist in the photograph is testing which muscle?
A. anterior deltoid.
B. middle deltoid.
C. supraspinatus.
D. upper trapezius.

B23 C6 2
23. A patient presents with pain and instability of the left foot/ankle secondary to poliomyelitis with more recent
development of progressive post-polio muscle atrophy. In this case, a plastic solid ankle-foot orthosis is an
appropriate prescription in order to:
A. control excessive amounts of knee flexion during swing.
B. maintain dorsiflexion throughout swing.
C. provide modest assistance to dorsiflexion while restricting plantar flexion.
D. restrict all movement.

B24 C1 1
24. A patient is referred to physical therapy following an anterior dislocation of the right shoulder. A possible positive
examination finding as the result of this trauma would be:
A. positive drop arm test.
B. positive Neer test.
C. weak deltoids.
D. weak rhomboids.

B25 C8 1
25. A 3 month-old infant is being examined at an early intervention program. The therapist is having difficulty flexing
the right upper extremity to remove the infants clothing. This task is made more difficult if:
A. the infant is turned into sidelying on the left.
B. the infant sits up with support of the head in a neutral position.
C. the infants head is turned to the left.
D. the infants head is turned to the right.

373406391.doc
Practice Questions 5

B26 C7 3
26. You have been asked to give an inservice presentation to staff nurses on safe guarding techniques. In order to best
prepare for this talk, you should:
A. provide a questionnaire to a random sampling of participants one week before the scheduled presentation.
B. provide a questionnaire to all participants two weeks before the scheduled session.
C. survey your audience at the scheduled session.
D. survey your audience just before the scheduled session.

B27 C2 3
27. The patient with left hemiplegia would be least likely to respond in therapy if the motor learning strategies
emphasized:
A. encouragement of the patient to slow down.
B. maximum use of demonstration and gesture.
C. maximum use of verbal cues.
D. simplification/restructuring of the environment including removal of all clutter.

B28 C3 2
28. A two-week old infant born at 27 weeks gestation with hyaline membrane disease is referred for a physical therapy
consult. Nursing reports that the child desaturates to 84% with handling and has minimal secretions at present.
The physical therapist should:
A. delegate to a physical therapist assistant for a maintenance program of manual techniques for secretion
clearance.
B. perform manual techniques for secretion clearance, 2 to 4 hours daily, to maintain airway patency.
C. provide suggestions to nursing for positioning for optimal motor development.
D. put the PT consult on hold as the child is too ill to tolerate exercise.

B29 C1 2
29. A thirty-four year-old male sustained a valgus stress to his left knee while skiing. His orthopedist found a positive
McMurrays test, and a positive Lachman stress test. He has been sent to physical therapy for conservative
management of his dysfunction. The sub-acute phase of physical therapy intervention should emphasize:
A. closed-chain functional strengthening of the quadriceps femoris and hamstrings emphasizing regaining
terminal knee extension.
B. closed-chain functional strengthening of the quadriceps femoris and hip abductors to promote regaining
terminal knee extension.
C. open-chain exercises of the hip extensors and hamstrings to inhibit anterior translation of the femur on the
tibia.
D. open-chain strengthening of the quadriceps femoris and hip adductors to inhibit anterior translation of the tibia
on the femur.

B30 C8 1
30. The most enthusiastic visual tracking by a neonate would be elicited by:
A. a black and white face, with a red nose, held approximately nine inches from the infants eyes, moved
horizontally.
B. a multicolored clowns face, held 12 inches away and moved horizontally.
C. a multicolored spinning top placed 15 inches from the infants eyes.
D. a red ring on a string, held six inches away from the infants eyes and moved horizontally.

B31 C2 2
31. A 14 year-old patient with traumatic brain injury has a convulsive seizure during a therapy session. She loses
consciousness and presents with tonic-clonic convulsions of all extremities. Your BEST response is to:
A. initiate CPR immediately and call for help to restrain her.
B. position in sidelying, check to see if the airway is open, and immediately call for emergency assistance.
C. position in supine with head supported with a pillow and wait out the seizure.
D. use straps to secure her limbs so she cant hurt herself.

373406391.doc
6 Practice Questions

B32 C7 2
32. A therapist wants to know whether NDT handling techniques produces an improvement in independent rolling that
lasts longer than 30 minutes. In this study rolling is the:
A. control variable.
B. dependent variable.
C. independent variable.
D. intervening variable.

B33 C3 1
33. A 45 year-old computer programmer, with no significant past medical history, presents to the emergency room
with complaints of fever, shaking chills and a worsening productive cough. He has chest pains over the posterior
base of his left thorax which is made worse on inspiration. Which of the following is NOT an appropriate physical
finding for this patient?
A. asymmetrical breathing.
B. crackles over the posterior aspect of his left thorax.
C. limited chest excursion.
D. slowed respiratory rate.

B34 C6 2
34. A patient is demonstrating genu valgum during standing and walking This problem can be effectively controlled
by prescribing a knee-ankle-foot orthosis with:
A. anterior knee cap strap.
B. posterior plastic shell.
C. pretibial and suprapatellar anterior bands.
D. quadrilateral brim.

B35 C4 1
35. You are called in to consult on a nursing home patient who, following a severe stroke and congestive heart failure,
is confined to bed and is immobile. During a skin inspection you note an area of erythema over her left sacrum
which persists longer than 50% of time pressure was applied to it. You determine that this is MOST likely:
A. an indurated decubitus ulcer with signs of maceration.
B. poor circulation secondary to end stage congestive heart failure.
C. Stage 1 ulcer formation.
D. Stage 2 ulcer formation.

B36 C5 3
36. Your patient strained the lower back muscles three weeks ago. The patient complains of pain (6/10) and you note
bilateral muscle spasm from T11-L4. You elect to apply interferential current. The BEST electrode configuration to
choose in this case would be:
A. four electrodes with current flow diagonal to the spinal column.
B. four electrodes with current flow perpendicular to the spinal column.
C. two electrodes with current flow parallel to the spinal column.
D. two electrodes with current flow perpendicular to the spinal column.

B37 C8 1
37. An infant has just begun to pull-to-stand through kneeling, still demonstrates plantar grasp in standing, and is
independent in sitting including all protective extension reactions. This infants chronological age is
approximately:
A. 5-6 months.
B. 7-8 months.
C. 9-10 months.
D. 12-14 months.

373406391.doc
Practice Questions 7

B38 C1 2
38. A sixty-five year old male retired bus driver has an increasing frequency of low back pain over the last 10 years.
He states that NSAIDs help to relieve his symptoms but there is always a nagging type pain. He reports significant
stiffness in the morning which dissipates by noon after exercising and walking his dog. Pain is exacerbated with
frequent lifting and bending activities as well as sitting for long periods. Physical therapy intervention should
emphasize:
A. joint mobilization, soft tissue mobilization, and flexion exercises.
B. modalities to reduce pain, joint mobilization, and lumbar extension exercises.
C. modalities to reduce pain, postural reeducation and dynamic stabilization exercises.
D. postural reeducation, soft tissue mobilization, and dynamic stabilization.

B39 C7 3
39. You are orienting a new physical therapy aide in transfer techniques. Your initial consideration should be to:
A. ask about previous work and other experiences related to transferring individuals.
B. determine specific goals for teaching the techniques.
C. give a computer simulated instructional program before actual hands on training takes place.
D. provide an organized series of talks dealing with patient safety during transfers.

B40 C7 2
40. A 77 year-old patient with diabetes and bilateral lower extremity amputation is to be discharged from an acute care
hospital two weeks post surgery. The incisions on his residual limbs are not healed and continue to drain. He is
unable to transfer because the venous graft sites in his upper extremities are painful and not fully healed. The
MOST appropriate discharge destination for this patient would be:
A. custodial care facility.
B. home.
C. rehabilitation hospital.
D. skilled nursing facility.

B41 C2 1
41. A 54 year-old man awoke one morning with drooping left facial muscles and excessive drooling. He was
recovering from a cold and had experienced an earache in his left ear during the previous 2 days. You suspect a
Bells Palsy which can be confirmed by examining:
A. corneal reflex and stretch reflexes of facial muscles.
B. taste sensation over the anterior tongue as well as motor function of facial muscles.
C. taste sensation over the posterior tongue as well as motor function of masseter muscle.
D. trigger points for pain, especially over the TMJ joint.

B42 C6 1
42. You assign a physical therapist assistant to ambulate a 72 year-old patient with Parkinsons disease. You instruct
the PTA to watch for:
A. an abnormally wide base of support.
B. decreased trunk rotation with shorter steps.
C. unsteady, uneven gait with staggering steps to one side.
D. wider strides and increased double support time.

B43 C3 1
43. A patient with congestive heart failure is on a regimen of diuretics and calcium channel blockers. The potential
side-effects of these medications that the physical therapist should be alert for include:
A. decreased electrolytes and electrical instability evidenced by increased arrhythmias.
B. gastrointestinal upset and extreme fatigue.
C. orthostatic hypotension and dizziness.
D. reflex tachycardia and unstable blood pressure.

373406391.doc
8 Practice Questions

B44 C1 2
44. A 42 year-old patient with a traumatic injury to her right hand had a flexor tendon repair to the fingers. Physical
therapy intervention following this type of repair should begin:
A. within a few days following surgery to preserve tendon gliding.
B. within a few days following surgery to allow for early initiation of strengthening exercises.
C. after the splint is removed in 2-3 weeks to allow full active range of motion of all affected joints.
D. after the splint is removed in 4-6 weeks to allow ample healing time for the repaired tendon.

B45 C8 1
45. An 89 year-old community dwelling elder fell in her home and suffered multiple fractures of her right arm
including Colles fracture of her right wrist, and humeral fracture of her right shoulder. She is hospitalized for open
reduction, internal fixation of her right radius. Which is NOT an expected finding with this patient?
A. decreased pain and tenderness at the fracture sites.
B. increased likelihood of developing complications like pneumonia.
C. mental confusion following hospitalization and surgery.
D. slower healing time with prolonged rehabilitation.

B46 C2 1
46. A patient exhibits bitemporal hemianopsia. In order to produce this condition, the lesion must be located at the:
A. left optic radiation.
B. left temporal lobe.
C. optic chiasm.
D. right parietal lobe.

B47 C6 1
47. Your patient is recovering from a left tibial amputation and complains of numbness and tingling affecting his
dorsal foot and big toe. This is driving him crazy because he knows the limb is gone. You recognize the source of
his discomfort is most likely pressure from residual limb wrapping affecting the:
A. common peroneal nerve.
B. medial calcaneal nerve.
C. sural nerve.
D. tibial nerve.

B48 C7 2
48. A therapist investigated the accuracy of pulse oximeter estimates during exercise. Correlational analysis measured
the strength of the relationship between two types of ear-probe-equipped pulse oximeters during heavy cycle
exercise under hypoxic conditions. The investigator found measured arterial oxyhemoglobin saturation (%HbO2)
levels to have a correlation of .89 at high saturation but only .68 at low saturation levels. The results of this study
suggest:
A. accuracy of the measurements increases at higher saturation levels.
B. both machines are highly accurate at all saturation levels.
C. both machines are only moderately accurate.
D. during heavy exercise, oxygen saturation levels should be interpreted cautiously.

B49 C1 1
49. A twenty-one year old college soccer player sustained a hyperextension knee injury when kicking the ball with his
other lower extremity. The patient was taken to the emergency room of a local hospital and was diagnosed with
knee sprain. He was sent to physical therapy the next day for aggressive rehabilitation. As part of the
examination to determine the type of treatment plan to implement, the therapist conducted the test shown in the
figure. Based on the test picture, the therapist is examining the integrity of the:
A. anterior cruciate ligament.
B. medial collateral ligament.
C. medial meniscus.
D. posterior cruciate ligament.

373406391.doc
Practice Questions 9

B50 C1 2
50. A 28 year-old male week-end athlete sustained a right shoulder injury when he was hit from behind while
throwing a football. His right upper extremity was flexed, abducted and externally rotated in preparation for
throwing when he was hit. Early physical therapy intervention should focus on:
A. AROM which emphasizes abduction and external rotation.
B. pendulum exercises.
C. strengthening of the middle trapezius muscle.
D. strengthening of the rotator cuff muscles.

B51 C5 1
51. You are to immerse an acutely sprained ankle into an ice water bath. You should tell the patient to FIRST expect:
A. aching, numbness, and burning, followed by intense cold.
B. burning, intense cold, and aching, followed by numbness.
C. intense cold, burning, and aching, followed by numbness.
D. numbness, aching, and intense cold, followed by burning.

B52 C8 3
52. Upon admission to a local hospital, a patient is given an Advanced Care Medical Directive to review and sign.
Guidelines for this document would NOT include:
A. an individual can refuse treatment even if it leads to death.
B. an individual must be mentally competent, with signing witnessed by two adults.
C. individual facilities modifying its format and implementation schedule.
D. patients, upon hospital admission, must be informed of their right to make decisions regarding their medical
care.

B53 C2 2
53. Your patient is recovering from stroke. At 4 months he is ambulating with a straight cane for household distances.
In the clinic, when you take his cane away and have him practice ambulating with no assistive device, you observe
recurvatum that increases as the session continues. The BEST strategy for you to take is to:
A. give him a KAFO to control the hyperextension and have him use a hemi walker.
B. give him a small based quad cane to improve his stability and have him practice AROM in supine.
C. practice isolated small range quadriceps eccentric control work in standing and continue with the straight
cane.
D. put him on a Cybex and work on increasing quadriceps torque output at higher loads and increasing speeds.

B54 C6 2
54. Your patient tells you that he is having difficulty going down ramps. He reports his knee wobbles and seems
unsteady. An appropriate intervention for this problem would be:
A. biofeedback training to reduce a knee extensor spasticity.
B. progressive resistance training for the quadriceps.
C. prolonged icing to reduce hamstring pain.
D. stretching via a posterior resting splint for tight plantar flexors.

B55 C7 3
55. It is illegal to release patient information without obtaining the patients consent to:
A. another healthcare provider involved in the care of the patient.
B. the insurance company that is paying for the patients treatment.
C. the patients employer when the condition is work-related.
D. the referring physician when there is a referral relationship.

B56 C6 3
56. With respect to the workers sitting postures in the Figure, the greatest reduction in lumbar spine compression
forces would be achieved by: (2002) The Kinematics of Sitting, Herman Miller Co., with permission
A. combining the usage of a lumbar support, armrests, and increase the angle between the seat pan and backrest
to between 90-110 degrees.
B. reclining the backrest of the chair at an angle between 90-110 degrees.
C. using a lumbar support.
D. using armrests on the chair.

373406391.doc
10 Practice Questions

B57 C1 1
57. A baseball pitcher has been sent to physical therapy with progressive posterior shoulder pain and weakness of the
shoulder abductors and lateral rotators. You notice muscle wasting superior and inferior to the scapular spine. The
patients problem is MOST LIKELY attributable to damage involving the:
A. long head of the biceps brachii.
B. scalene muscles.
C. spinal accessory nerve.
D. suprascapular nerve.

B58 C7 3
58. Nursing homes that receive Medicare reimbursement for eligible residents are required by law to provide for
rehabilitation services including physical therapy based on:
A. diagnostic categories (DRGs).
B. needs assessment performed by a physical therapist.
C. referral from a physician.
D. referral from the nurse case manager.

B59 C2 2
59. A 62 year-old patient is recovering from surgical resection of an acoustic neuroma. She presents with symptoms of
dizziness, vertigo, horizontal nystagmus, and postural instability. To address these problems, her physical therapy
plan of care should incorporate:
A. Hallpike exercises to improve speed in movement transitions.
B. prolonged bed rest to allow vestibular recovery to occur
C. repetition of movements and positions that provoke dizziness and vertigo.
D. strengthening exercises focusing on spinal extensors.

B60 C6 1
60. Your patient is beginning her ambulation training with a right above-knee prosthesis. You notice that during early
swing the heel rises excessively. Possible causes are:
A. amputee pain or discomfort.
B. inadequate knee friction or too little tension in the extension aid.
C. too much knee friction or too little tension in the extension aid.
D. too much tension in the extension aid.

B61 C4 2
61. You are treating an elderly 93 year-old in the home environment. When you arrive you note that he is somewhat
confused and his color and skin turgor is poor. He reports that he has had an intestinal bug the last few days with
frequent vomiting and diarrhea. Your BEST course of action is to:
A. cancel therapy for today and carefully document your findings in his chart.
B. give him some water to drink and report your findings to his physician immediately.
C. monitor his vital signs as you get him up and ambulating.
D. notify the family, request they keep him company until he gets over his illness.

B62 C3 2
62. For patients with coronary artery disease participating in a cardiac rehabilitation class an indicator that the exercise
intensity is too great and should be reduced would be:
A. a systolic blood pressure greater than 140 mm Hg or diastolic BP greater than 80 mm Hg.
B. appearance of a PVC on the ECG.
C. greater than 1 mm ST segment depression, horizontal or downsloping.
D. peak exercise HR greater than 140.

B63 C8 2
63. An infant who was 33 weeks gestational age at birth and is now 3 weeks chronological age demonstrates colic. In
this case, the BEST intervention the physical therapist could teach the mother is:
A. fast vestibular stimulation.
B. neutral warmth.
C. stroking and tapping.
D. visual stimulation with a colored object.

373406391.doc
Practice Questions 11

B64 C2 1
64. You are treating a 24 year-old man who exhibits Brown-Sequard syndrome as a result of a gun shot wound. Your
examination should reveal:
A. ipsilateral weakness and loss of position sense and vibration below the lesion level with contralateral loss of
pain & temperature sensation.
B. loss of motor function and pain and temperature sensation with preservation of light touch and position sense
below the level of the lesion.
C. loss of upper extremity function, (cervical tract involvement) with preservation of lower extremity function
(lumbosacral tract involvement).
D. sparing of tracts to sacral segments with preservation of perianal sensation, and active toe flexion.

B65 C7 2
65. A 62 year-old man is recovering from stroke and presents with moderate impairments of the left upper and lower
extremities. Your goal today is to instruct him in a stand-pivot transfer to his affected side so he can go home on a
weekend pass. His wife is attending todays session and will be assisting him on the weekend. Your BEST choice
for instruction is:
A. demonstrate the task, then have the caregiver practice with the patient.
B. demonstrate the task, then practice with the patient.
C. practice first with the caregiver, then with the patient.
D. practice first with the patient, then with the caregiver.

B66 C7 3
66. Your patient is a 78 year-old male currently in an acute care facility recovering from fractures of the spine and hip.
His Medicare Part A benefits have come to an end and he is soon to be discharged. Medicare Part B will NOT
cover:
A. durable medical equipment needed for the patient in his home.
B. home health agency rehabilitation services.
C. outpatient therapy services in a private therapy practice.
D. outpatient therapy services in an acute care facility.

B67 C6 1
67. When using a patellar-tendon bearing prosthesis, a patient will experience excessive knee flexion in early stance if
the:
A. foot is inset too much.
B. foot is outset too much.
C. socket is aligned too far back or tilted posteriorly.
D. socket is aligned too far forward or tilted anteriorly.

B68 C3 2
68. A chronic smoker developed carcinoma of the lung. Following a right pneumonectomy, post-operative physical
therapy intervention should include:
A. breathing exercises to both sides of the thorax to maintain adequate aeration.
B. percussion in all postural drainage positions.
C. shaking in all postural drainage positions.
D. sustained maximal inspiration training with an incentive spirometer.

B69 C6 2
69. As part of your plan of care, you have to order a wheelchair for a 32 year-old patient with a T9-10 spinal cord
injury. Her wheelchair wont need:
A. a high back.
B. a low back.
C. removable arm rests.
D. swing-away footrests.

373406391.doc
12 Practice Questions

B70 C1 2
70. A patient complains of persistent wrist pain after painting her house three weeks ago. The patient demonstrates a
positive Finkelsteins Test, pain and swelling at the anatomical snuff box, and decreased pinch strength. An
effective plan of care should include:
A. desensitization techniques to reduce symptoms of reflex sympathetic dystrophy.
B. gentle active assistive exercises to regain function after sustaining a gamekeepers thumb injury.
C. iontophoresis to reduce inflammation at the carpal tunnel.
D. phonophoresis to reduce inflammation of the tendinous sheaths.

B71 C8 2
71. An adaptive wheelchair for a child with moderate spastic quadriplegic cerebral palsy would include:
A. a pommel to keep hips abducted.
B. movable chairback to allow for extension of the hips and trunk.
C. movable footrests to allow ankles to plantarflex.
D. pads on the seat to keep hips adducted.

B72 C2 1
72. A 73 year-old has persistent balance dysfunction and a history of recent falls (2 in the last 3 months). In your
initial session it is crucial to examine:
A. cardiovascular endurance during a 6 minute walking test.
B. level of dyspnea during functional transfers.
C. sensory losses and sensory organization of balance.
D. spinal musculoskeletal changes secondary to degenerative joint disease.

B73 C7 3
73. A patient with adhesive capsulitis of the shoulder sustains a fracture of the shoulder during treatment provided by
the PTA. The fracture occurred while the PTA was mobilizing the shoulder joint, which was part of the plan of
care established by the PT. After the incident, the PTA told the PT that she was not familiar with the mobilization
techniques to the shoulder. Responsibility in this case falls on:
A. both the PT and the PTA are responsible for establishing effective communication regarding the skills and
competencies of the PTA.
B. neither the PT nor the PTA are responsible for the fracture, it is an accepted risk associated with joint
mobilization.
C. the PT who is solely responsible for assessing the competence of the PTA under their supervision.
D. the PTA who is responsible for informing their supervising PT whenever they are unfamiliar or uncomfortable
with any treatment procedure.

B74 C7 3
74. A new patient arrives at the physical therapy department for an initial examination, evaluation and treatment of a
second degree ankle sprain. The therapist is busy writing a discharge summary when the patient with the ankle
sprain arrives for his scheduled appointment. The physical therapist assistant on staff knows the patient as he had
treated the patient previously for a similar injury. The physical therapist should:
A. ask the physical therapist assistant to evaluate the patients range of motion and functional status to speed up
the evaluation process.
B. ask the physical therapist assistant to perform a complete evaluation.
C. complete the written component of the discharge summary at another time and evaluate this new patient.
D. tell the patient to take a seat and wait until the discharge summary is completed.

B75 C1 1
75. During a postural screen for a patient complaining of low back pain you notice that the knees are in genu
recurvatum. Possible contributory postures would include:
A. ankle dorsiflexion and hip abduction.
B. ankle plantarflexion and anterior pelvic tilt.
C. forefoot varus and posterior pelvic tilt.
D. lateral tibial torsion and anterior pelvic tilt.

373406391.doc
Practice Questions 13

B76 C1 1
76. After treating a patient for trochanteric bursitis for one week, the patient has no resolution of pain and is
complaining of problems with gait. After re-examination you find weakness of the quadriceps femoris, and altered
sensation at the greater trochanter. You suspect the problem is:
A. degenerative joint disease of the hip.
B. L4 nerve root compression.
C. L5 nerve root compression.
D. sacroiliac dysfunction.

B77 C4 1
77. You are treating a patient in chronic renal failure. You need to schedule his physical therapy sessions around his
dialysis which he receives 3 mornings a week. He is also hypertensive and requires careful monitoring during his
ambulation program. Your BEST approach is:
A. place him supine before taking his BP, using the shunt arm.
B. take his BP every minute during exercise, using his shunt arm.
C. take his BP pre and post exercise, using his non-shunt arm.
D. wait until he stops walking before taking his BP in the seated position.

B78 C6 2
78. A patient is recovering from a fracture of both tibia and fibula in the right lower leg which has been casted. The
referral is for gait training, nonweightbearing on right lower extremity. The safest gait pattern for this patient to use
is a:
A. four-point.
B. swing-to.
C. three-point.
D. two-point.

B79 C7 2
79. A therapist wishes to determine the effectiveness of TENS on the relief of pain in a group of 20 patients with
phantom limb pain. She recruits her patients over a 2 year period. All receive a 6 week intervention. She finds that
12 patients with phantom pain got pain relief while 8 had no relief. The BEST conclusion that can be reached is:
A. 40% of patients with phantom pain do not benefit from TENS.
B. since a no treatment group was not used for comparison, no conclusions can be reached.
C. since a quasi-experimental design was used, the treatment effectiveness of TENS is established.
D. TENS has a 60% rate of effectiveness.

B80 C3 2
80. Following a post-myocardial infarction, a patient is a new admission to your Phase 3 cardiac rehabilitation
program. During the initial exercise session his ECG responses are continuously monitored via radio telemetry.
You notice three PVCs occurring in a run with no P wave. The MOST appropriate action to take is to:
A. continue the exercise session but monitor closely.
B. have him sit down and rest for a few minutes before resuming exercise.
C. modify the exercise prescription by decreasing the intensity.
D. stop the exercise and notify the physician immediately.

B81 C2 1
81. A 63 year-old patient with left hemiplegia is able to recognize his wife after she is with him for awhile and talks to
him, but he is unable to recognize the faces of his children when they come to visit. The children are naturally very
upset by their fathers behavior. The BEST explanation for his problem is:
A. anosognosia.
B. ideational apraxia.
C. somatognosia.
D. visual agnosia.

373406391.doc
14 Practice Questions

B82 C8 1
82. You have received a referral to evaluate the fall risk potential of an 82 year-old community dwelling elder. He has
fallen three times in the last four months with no history of fall injury except for minor bruising. He currently
presents with an increased fear of falling. The factor LEAST likely to contribute to his fall risk is:
A. dementia or depression.
B. drugs that produce essential tremor.
C. drugs that result in orthostatic hypotension.
D. psychotropic drugs.

B83 C1 2
83. After examining a patient with foot pain, the therapist finds the following positive findings: reproduction of
symptoms in weight-bearing and running on a treadmill, pes planus and pain with palpation at the distal aspect of
the calcaneus. Late sub-acute management, when pain is manageable/reduced, would include:
A. examination for an orthosis.
B. modalities to reduce pain and inflammation.
C. strengthening of ankle dorsiflexors.
D. stretching of plantar fascia.

B84 C6 3
84. A 22 year-old patient with a complete T10 paraplegia resulting from a spinal cord injury is ready to begin
community wheelchair training. Today your goal is to teach him how to do a wheelie so he can manage curbs.
Your BEST training strategy is to instruct the patient to:
A. grasp the handrims posteriorly and pull them forward abruptly and forcefully.
B. lean backward while moving the hands slowly backward on the rims.
C. place his hand on the top of the handrims to steady the chair while he throws his head and trunk forward.
D. throw his head and trunk backward to rise up on the large wheels.

B85 C3 1
85. A physical therapist should be alert to recognize the signs and symptoms associated with the onset of aspiration
pneumonia. Of the following, the patient MOST susceptible to develop this form of pneumonia is one with:
A. a circumferential burn of the thorax associated with significant pain.
B. a complete spinal cord lesion at T2 with diminished coughing ability.
C. amyotrophic lateral sclerosis with dysphagia and diminished gag reflex.
D. severe scoliosis with compression of internal organs.

B86 C6 2
86. A patient demonstrates quadriceps weakness (4- /5) and difficulty descending stairs. The BEST intervention to
regain functional strength in the quadriceps is:
A. isokinetic exercise, at 36 degrees/second.
B. maximum isometric exercise, at 45 and 90 degrees of knee extension.
C. partial squats progressing to lunges.
D. progressive resistance exercises, 70% 1 RM, 3 sets of 10.

B87 C2 1
87. Independent community ambulation as the primary means of functional mobility is a realistic functional
expectation for a patient with the highest level of spinal cord injury at:
A. high lumbar (T12-L1).
B. low lumbar (L4-L5).
C. low thoracic (T9-10).
D. midthoracic (T6-T9).

373406391.doc
Practice Questions 15

B88 C5 2
88. Your patient had multiple fractures of both hands and wrists as a result of a mountain bike accident. Now five
weeks later, the patient currently has vertigo, limited wrist and finger motion, and dry scaly skin over the involved
areas. The physical agent MOST appropriate to select in this case would be:
A. functional electrical stimulation, seated.
B. hot packs with the patient positioned in a recumbent position.
C. infrared with the patient positioned supine on a treatment table.
D. paraffin with the patient positioned in a recumbent position.

B89 C6 2
89. Your patient is 82 years-old and demonstrates a history of recent falls (two in the last two months) and mild
balance instability. Your referral is to examine the patient and recommend an assistive device as needed. Based on
the patients history, it would be BEST to select a:
A. a folding reciprocal walker.
B. front wheel rolling walker that folds.
C. hemi walker.
D. standard, fixed frame walker.

B90 C1 3
90. Following surgery, a patient responds with a stiff pelvis and limited pelvic/lower trunk mobility. You elect to use
sitting exercises on a Swiss ball to correct these impairments. In order to improve lower abdominal control, the
ball would have to move:
A. backward, producing anterior tilting of the pelvis.
B. backward, producing posterior tilting of the pelvis.
C. forward, producing anterior tilting of the pelvis.
D. forward, producing posterior tilting of the pelvis.

B91 C2 2
91. A patient who has been in a coma for 8 weeks is newly admitted to an extended care facility. Your focus is to
involve the family in the plan of care since both parents are faithful in visiting on daily basis. You decide it is
MOST important to:
A. demonstrate PROM techniques, offering corrections as they perform the exercises for you.
B. give them written instructions detailing his PROM exercises.
C. instruct them to talk to the patient but restrict them from all hands on assistance.
D. tell them their participation will come once the patient becomes conscious, right now the professional staff
must do his exercises.

B92 C2 1
92. A patient presents with weakness of the entire right half of the face. She is unable to completely close her eye.
Sensation is normal. She tells you that she recently had the flu and has recovered slowly from this illness. You
suspect:
A. abducens nerve pathology: the right eye pulls inward.
B. facial nerve pathology: Bells palsy.
C. oculomotor nerve pathology: nystagmus.
D. trigeminal nerve pathology: tic douloureux.

B93 C3 2
93. A patient with bacterial pneumonia has crackles and wheezes at her left lateral basal segment and decreased breath
sounds throughout. She in on 4 liters of oxygen by nasal cannula which brings her SaO2 to 90%. Respiratory Rate
is 28. The intervention that would be INAPPROPRIATE to use in this case is:
A. breathing exercises encouraging expansion of the left lateral basilar thorax.
B. percussion to the appropriate area on the left lateral basilar thorax.
C. shaking over the appropriate area on the left lateral basilar thorax.
D. standard postural drainage for the lateral basal segment, left lower lobe.

373406391.doc
16 Practice Questions

B94 C8 1
94. You receive a referral to see an elderly patient in the ICU recovering from a severe case of pneumonia. You
recognize his disorientation is due to delirium rather than dementia because:
A. he demonstrates persistent personality changes.
B. he has hallucinations throughout the day.
C. his level of arousal is significantly depressed.
D. this cognitive symptoms are intermittent and come on suddenly, often at night.

B95 C7 2
95. A comparison of the effects of exercise in water, on land, or combined on the rehabilitation outcome of patients
with intra-articular anterior cruciate ligament reconstructions revealed that less joint effusion was noted after 8
weeks in the water group. An appropriate statistical test to compare the girth measurements of the three groups is:
A. analysis of covariance.
B. analysis of variance.
C. chi square.
D. Spearman rho.

B96 C2 2
96. A 64 year-old man has a recent history of strokes (2 in the past 4 months). He demonstrates good return of his right
lower extremity and is walking with a straight cane. You are concentrating on improving his balance and
independence in gait. Unfortunately his speech recovery is lagging behind his motor recovery. He demonstrates a
severe fluent aphasia. The BEST strategy during therapy is:
A. consult with the speech therapist to verbally interpret your instructions.
B. involve the family for all treatment sessions and have them translate for you.
C. utilize demonstration and gesture to get the idea across of what you want him to do.
D. utilize verbal cues, emphasizing consistency and repetition.

B97 C6 1
97. An individual is walking with an above-knee prosthesis and demonstrates terminal swing impact. You suspect:
A. insufficient knee friction.
B. the hip flexors are weak.
C. the prosthesis is externally rotated.
D. too little tension in the extension aid.

B98 C4 2
98. A 52 year-old man was burned over 40% of his body in an industrial accident. He has full thickness burns over his
anterior trunk and neck and superficial partial thickness burn over his shoulders. The MOST appropriate splinting
procedures to stabilize this patient out of positions of common deformity include:
A. arm slings that fasten the arms to the chest.
B. hand splints (intrinsic minus position) with plastic jacket for the trunk.
C. plastic cervical orthosis and axillary splints (airplane position).
D. soft cervical collar that allows some neck flexion.

B99 C1 1
99. A 16 year-old female, seen in the emergency room, was placed in a short leg cast following a severe right-sided
Grade III ankle sprain. She is immediately referred to physical therapy for gait training using crutches. She
complains that her right great toe is numb and she cannot lift her toes. This problem MOST LIKELY is a result of:
A. anxiety and hyperventilation in anticipation of using crutches.
B. compression of the common peroneal nerve.
C. compression of the medial plantar nerve.
D. compression of the posterior tibial artery resulting in vascular insufficiency.

B100 C2 3
100. Which intervention would be LEAST likely used to improve left-sided neglect in a patient with left hemiplegia?
A. bridging with arms at sides.
B. hooklying, lower trunk rotation, rhythmic initiation.
C. rolling, supine to sidelying on right, using PNF lift pattern.
D. sitting, with hands forward resting on large ball, weight shifting moving ball to left.

373406391.doc
Practice Questions 17

B101 C7 3
101. A 20 year-old patient with traumatic brain injury who has been receiving in-patient physical therapy for the past
two months has not demonstrated improvement in functional status for a considerable period of time. The patient
is covered by Medicare. You have informed both the referring physician and the patients family of his lack of
progress. The family insists that you continue to treat the patient, and the physician continues to refer the patient
for more treatment and certifies the necessity of the care. You should:
A. continue to provide the care both the family and the referring physician demand; it is the physicians
responsibility to determine the appropriateness of physical therapy.
B. give the patients family Medicare notification of noncoverage information, and carefully explain it, and their
options, which could include paying for the care out-of-pocket.
C. modify treatment goals in a manner that will allow you to demonstrate that the treatment is achieving progress
towards reasonable goals.
D. refer the patient to another clinic that you believe will be willing to continue treatment despite lack of
functional improvement.

B102 C3 2
102. A patient with COPD reports to his fourth outpatient pulmonary rehabilitation session complaining of nausea,
gastric upset and feeling jittery. The patient reports no change in pulmonary symptoms. The physical therapist
records the following set of vital signs: temperature 98.6F, heart rate 110 beats/min, and irregular blood pressure
150/86, respiratory rate 20. Breath sounds show no change from baseline. The therapist checks the medical record
and finds that the patient has no history of gastric disease. He is presently taking theophylline, Ventolin and
Azmacort. The physical therapist should:
A. call the patients physician due to signs of theophylline toxicity.
B. have the patient call the physician when he returns home due to signs of increased pulmonary dysfunction.
C. have the patient use his Ventolin to improve respiratory status.
D. send the patient home and reschedule for another day.

B103 C8 1
103. You are treating a one year-old child with Down Syndrome at home and you notice a decrease in strength in the
extremities. DTRs are 1+. Your immediate thought is:
A. hypertonia and decreased strength are to be expected with Down Syndrome.
B. hypotonia and decreased strength are to be expected with Down Syndrome.
C. the parents are not performing the home exercise program.
D. there may be a dislocation at C1/C2 due to ligamentous laxity.

B104 C2 1
104. Which of the following causative factors is NOT likely to produce shoulder pain in hemiplegia?
A. flaccid paralysis with loss of seating action of rotator cuff muscles.
B. passive range of motion while emphasizing scapulohumeral rhythm.
C. spastic retraction with depression and downward rotation of scapula.
D. traction and gravitational forces acting on a depressed, downwardly rotated scapula.

B105 C6 1
105. While gait training a patient recovering from a cerebral vascular accident, you observe the knee on the affected
side going into recurvatum during stance phase. The MOST LIKELY cause of this deviation can be attributed to:
A. severe spasticity of the hamstrings or weakness of the gastrocnemius-soleus.
B. weakness of both the gastrocnemius-soleus and pretibial muscles.
C. weakness of the gastrocnemius-soleus or spasticity of the pretibial muscles.
D. weakness or severe spasticity of the quadriceps.

B106 C4 2
106. A patient with an ulcer is treated with a whirlpool to assist in debridement and wound cleansing. The laboratory
report comes back positive for active local infection. You instruct the physical therapist assistant to drain and clean
the whirlpool. Disinfection of the whirlpool can best be achieved with:
A. boiling water.
B. povidone-iodine.
C. silver nitrate solution.
D. ultraviolet exposure.

373406391.doc
18 Practice Questions

B107 C7 2
107. A therapist has completed a study investigating the relationship between ratings of perceived exertion (RPE) and
type of testing modality: arm ergometry versus leg ergometry. She finds a correlation 0.59 with the arm testing
while the correlation is 0.79 with the leg testing. She determines:
A. both arm and leg ergometry are highly correlated with RPE.
B. both arm and leg ergometry are only moderately correlated with RPE.
C. leg ergometry is highly correlated with RPE while arm ergometry is only moderately correlated.
D. the common variance of both types of testing is only 22%.

B108 C1 2
108. A 67 year-old male has a diagnosis of left knee degenerative joint disease. He complains of left-sided knee pain of
two months duration. He has been followed by outpatient physical therapy for three weeks. The patient feels his
condition is worsening as pain has increased during weight bearing activities and he can no longer fully extend his
left knee. Examination findings include: increased swelling, decreased knee AROM into extension, and an antalgic
gait. The physical therapist should:
A. continue physical therapy for another two weeks because there is uncertainty if the patient understands or is
complying with the home exercise program.
B. continue therapy for another week to ensure that all interventions have been attempted and then return the
patient to the referring physician.
C. immediately return the patient to the referring physician with documentation indicating that treatment was
ineffective.
D. tell the patient to see an orthopedic surgeon for possible immediate surgical intervention.

B109 C5 3
109. While playing volleyball two months ago, a 28 year-old had an inversion sprain of the calcaneofibular and anterior
talofibular ligaments of the right ankle. The ankle is still painful, very limited in motion and slightly tender to the
touch. As part of your intervention, ultrasound treatment parameters should consist of:
A. continuous US @ 1 MHz.
B. continuous US @ 3 MHz.
C. pulsed US @ 1 MHz.
D. pulsed US @ 3 MHz.

B110 C2 3
110. In Neurodevelopmental Treatment (NDT) of the patient recovering from stroke, therapy would NOT include:
A. facilitation of early movement in synergistic patterns followed quickly by movement patterns out-of-synergy.
B. facilitation of selective movement control out of synergistic patterns.
C. functional activities emphasizing reintegration of the hemiplegic side.
D. reduction of spasticity and abnormal reflex activity through positioning and handling techniques.

B111 C8 2
111. A 4 year-old child with moderate spastic diplegia is referred to physical therapy for an adaptive equipment check.
In this case, which equipment is NOT indicated?
A. bilateral KAFOs.
B. posture control walker (posterior walker).
C. prone stander
D. tone reducing AFOs.

B112 C6 2
112. A 67 year-old patient is recovering from a left CVA. He is wearing a plastic knee-ankle-foot orthosis to stabilize
his right foot. During gait analysis you observe lateral trunk bending toward the right as he bears weight on the
right leg at midstance. The BEST choice to correct this problem is:
A. provide a lift on the shoe of the involved leg.
B. strengthen hamstrings on the right side.
C. strengthen hip flexors on the right side.
D. strengthen the hip abductors on the right side.

373406391.doc
Practice Questions 19

B113 C1 2
113. A ten year-old male who plays catcher on his baseball team complains of bilateral knee pain which is exacerbated
with forceful quadriceps contraction. He has also noticed pain and swelling at the distal attachment of the patellar
tendon. Effective early physical therapy intervention should include:
A. AROM exercises to prevent contracture.
B. casting followed by decreased loading of the knee.
C. decreased loading of the knee by the quadriceps femoris muscle.
D. modalities to decrease inflammation.

B114 C3 2
114. An 83 year-old patient has a decubitus ulcer of 3 months duration on his lateral ankle. The ankle is swollen, red,
and painful with a moderate to high amount of wound drainage (exudate). The BEST dressing for this wound is:
A. calcium alginate dressings.
B. gauze dressings.
C. hydrogel dressings.
D. semipermeable film dressings.

B115 C1 1
115. A thirty-five year-old male was diagnosed with a bulging disc at the right L5-S1 spinal level without nerve root
compression. The functional loss he would MOST likely suffer from is:
A. centralized gnawing pain with loss of postural control during lifting activities.
B. centralized gnawing pain with uncompensated gluteus medius gait.
C. radicular pain to the right great toe with a compensated gluteus medius gait.
D. radicular pain to the right great toe with difficulty sitting for long periods.

B116 C2 2
116. High level management of an individual recovering from traumatic brain injury who demonstrates Rancho Level
Cognitive Function VII should focus on the avoidance of Robot syndrome. The best intervention strategy in this
case is represented by:
A. involving the patient in decision making, emphasizing safety and independent performance.
B. providing a high degree of environmental structure.
C. providing assistance for guided movements during all movement tasks.
D. providing maximum supervision to ensure successful performance.

B117 C6 2
117. To correct for the problem of a forward festinating gait in a patient with Parkinsons disease, you could:
A. increase cadence.
B. increase stride length.
C. use a heel wedge.
D. use a toe wedge.

B118 C6 3
118. A patient presents with difficulty with fast movement speeds and fatigues easily. You decide on a strength training
program that specifically focuses on improving fast-twitch fiber function. The optimal exercise prescription to
achieve this goal is:
A. high intensity workloads for long durations.
B. high intensity workloads for short durations.
C. low intensity workloads for long durations.
D. low intensity workloads for short durations.

B119 C2 1
119. You have been called to the traumatic brain injury unit to examine a new patient. The patient is highly agitated and
combative with the nursing staff. You assess his current state as being one of high arousal. The autonomic changes
you would expect to see include:
A. decreased responsiveness to sensory stimulation and lethargy.
B. hyperactive stretch and cutaneous reflexes, exaggerated pupillary reflexes.
C. increased HR and RR along with dilated pupils.
D. slowed HR with constriction of pupils, hypotension.

373406391.doc
20 Practice Questions

B120 C8 2
120. A physical therapy functional goal for a 5 year-old child with a very high lumbar lesion myelomeningocele and
minimal cognitive involvement would be;
A. ambulation for exercise with a reciprocating gait orthosis and Lofstrand crutches.
B. community ambulation with a reciprocating gait orthosis and Lofstrand crutches.
C. community ambulation with HKAFOs and Lofstrand crutches.
D. physiological ambulation with KAFOs and rollator walker.

B121 C7 2
121. A patient with multiple sclerosis is part of a national study testing the effectiveness of a new medication. The
patient reports that the pill she is taking makes her feel much better and allows her to move easier. At the
conclusion of the study it is revealed that she was part of the control group. Her responses are MOST likely due to:
A. hawthorne effect.
B. placebo effect.
C. pretest-treatment interference.
D. sampling bias.

B122 C3 1
122. You are reading a recent report of arterial blood gas analysis with the following values: Fraction of inspired
oxygen (FiO2) - 0.21, PaO2 - 53 mmHg, PaCO2 - 30 mmHg, pH - 7.48, Bicarbonate ion - 24 mEq/l This would
indicate that your patient is in:
A. metabolic acidosis.
B. metabolic alkalosis.
C. respiratory acidosis.
D. respiratory alkalosis.

B123 C4 3
123. A 32 year-old patient is referred to your cardiac exercise group following a mild myocardial infarction. From her
intake questionnaire you learn she also has Type I (IDDM) diabetes, controlled with twice daily insulin injections.
In order to minimize the risk of a hypoglycemic event during exercise, you should have the patient:
A. avoid exercise during periods of peak insulin activity.
B. exercise daily for 40-50 minutes to achieve proper glucose control.
C. have the patient decrease her carbohydrate intake for 2 hours before the exercise session.
D. monitor blood glucose levels carefully every week during the rehabilitation program.

B124 C8 2
124. An 85 year-old resident of a community nursing home is diagnosed with organic brain syndrome, Alzheimers
type. In formulating a plan of care, it is important to understand that she:
A. can usually be trusted to be responsible for her daily care needs.
B. can usually be trusted with transfers with appropriate positioning of the wheelchair.
C. is more likely to remember current experiences then past ones.
D. will likely be resistant to activity training if unfamiliar activities are used.

B125 C2 3
125. A patient is having difficulty learning how to transfer from mat to wheelchair. He just cant seem to get the idea of
how to coordinate this movement. In this case, the MOST effective use of feedback during early motor learning
should:
A. focus on guided movement and proprioceptive inputs.
B. focus on knowledge of performance and proprioceptive inputs.
C. focus on knowledge of results and visual inputs.
D. provide feedback only after a brief (5 second) delay.

373406391.doc
Practice Questions 21

B126 C6 2
126. A 52 year-old women presents with pain and paresthesias affecting the third and fourth toes of her left foot. She
typically wears shoes with 3 inch heels and pointed toes. You suspect she is presenting with metatarsalgia. The
BEST intervention is a:
A. pad over the transverse arch.
B. pad positioned distal to the metatarsal heads.
C. scaphoid pad.
D. Thomas heel.

B127 C1 2
127. A 38 year-old female developed right throbbing shoulder pain after painting her kitchen. Passive and active
glenohumeral motions increase pain. The BEST initial intervention for this acute shoulder condition is:
A. correction of muscle imbalances to allow healing of right shoulder supraspinatus tendinitis.
B. modalities to reduce pain and inflammation as the result of subdeltoid bursitis.
C. rotator cuff strengthening exercises to allow ADL function following biceps tendinitis.
D. stretching of the pectoralis minor muscle following acromioclavicular joint inflammation.

B128 C4 1
128. A 22 year-old male has a one year history of AIDS. The case worker reports a gradual increase in difficulty with
instrumental ADLs. A referral to P.T. is initiated. The most likely CNS deficits from AIDS are:
A. alterations in memory, confusion, and disorientation.
B. gait disturbances resulting from progressive rigidity and tremor.
C. increasing paralysis eventually leading to quadriplegia.
D. pronounced sensory loss resulting in sensory ataxia.

B129 C7 3
129. A new graduate physical therapist has an appointment for a job interview with Human Resources at a large
teaching hospital. She is well dressed, has a professional typewritten resume, and is prompt for her 2:00 p.m.
appointment. The interviewer should AVOID discussing:
A. health insurance, continuing education, and vacation days.
B. information regarding advantages of working in a large teaching hospital as well as disadvantages regarding
the job.
C. the applicants marital status.
D. work hours associated with the position.

B130 C8 3
130. You are working with a two year-old child with Down Syndrome who frequently uses a W sitting position. You are
explaining to the parents why this position should be discouraged. The main reason to discourage W sitting in this
young child is that it may:
A. cause femoral anteversion and knee strain.
B. decrease abnormally low tone.
C. delay the development of normal sitting.
D. increase abnormally high tone.

B131 C2 1
131. A 92 year-old patient is being examined for signs of stroke. When you test two-point discrimination on his right
hand, he is unable to tell you whether you are touching him with one or two points. You determine that there is
impaired function in the:
A. anterior spinothalamic tract or thalamus.
B. dorsal column/lemniscal pathways or somatosensory cortex.
C. lateral spinothalamic tract or somatosensory cortex.
D. spinal lemniscus or ventral posterolateral nucleus of the thalamus.

373406391.doc
22 Practice Questions

B132 C3 2
132. A 75 year-old outpatient has a history of two myocardial infarctions and one episode of recent congestive heart
failure. He also had claudication pain in the right calf during his exercise tolerance test. An INITIAL exercise
prescription that best deals with his problems is walking:
A. five times a week using continuous training, for 60 minutes.
B. several times a week using interval training, for 10 to 15 minute periods.
C. three times a week using continuous training, for 40 minute sessions.
D. three times a week using interval training, for 10 to 15 minutes periods.

B133 C7 3
133. A valid informed consent for research purposes should include all of the following elements EXCEPT:
A. a statement ensuring the subjects commitment to participate for the duration of the study.
B. all potential benefits of participation.
C. all reasonable and foreseeable risks and discomforts.
D. an understandable explanation of the purpose and procedures to be used.

B134 C4 1
134. During pregnancy, the presence of the hormone relaxin can lead to abnormal joint hypermobility and pain, MOST
frequently affecting the:
A. carpal and metacarpal joints.
B. lumbosacral joints.
C. sacroiliac joints.
D. spinal facet joints.

B135 C5 3
135. Your patient has severe low back pain as a result of chopping wood three weeks ago. The patient has been
receiving ultrasound and strengthening exercises to the low back for two weeks. You have also been applying
TENS and decide to have the patient use TENS at home. As a safety precaution, it is important to instruct the
patient to perform a daily check of the:
A. electrodes and electrode jacks.
B. electrodes and leads.
C. skin and electrodes.
D. skin and leads.

B136 C6 3
136. A tilt-in-space wheelchair has been ordered for a patient with C4 quadriplegia. This type of chair is ordered to:
A. facilitate handgrip and propulsion.
B. improve leg position and lower extremity edema.
C. improve positioning for pressure relief or for patients with extensor spasms.
D. improve the patients ability to relieve pressure and transfer independently.

B137 C8 2
137. An 82 year-old female has low vision. She recently returned home from a 2 week hospitalization for stabilization
of her diabetes. Your goal is to mobilize her and increase her ambulation level and safety. The BEST intervention
strategy for this patient is to:
A. color code stairs with pastel shades of blue and green to highlight steps.
B. keep window shades wide open to let in as much light as possible.
C. practice walking by having her look at her feet at all times.
D. practice walking in areas of high illumination and low clutter.

B138 C3 1
138. Your patient has a history of angina pectoris and limited physical activity. As you are supervising him in his
second exercise class you suspect that his angina is unstable and may be indicative of a preinfarction state. You
determine this by the presence of:
A. angina increasing in intensity and unresponsive to the nitroglycerin or rest.
B. angina responding quickly to rest and interval rather than continuous training.
C. arrhythmias increasing in frequency, especially atrial arrhythmias.
D. delayed cessation of pain following the administration of nitroglycerin.

373406391.doc
Practice Questions 23

B139 C2 3
139. Your patient suffered a left CVA which has left him hemiparetic on the right side. In his current stage of recovery,
he demonstrates strong and dominant hemiplegic synergies in his leg. Which activity would NOT be helpful to
break up these synergies?
A. assuming the bridging position.
B. foot tapping in a sitting position.
C. rolling from the hooklying position using lower extremity D1 flexion PNF pattern.
D. weight shifts in kneeling.

B140 C7 3
140. All of the following criteria would designate a patient as homebound and allow for home physical therapy services
to be approved by Medicare with the EXCEPTION of:
A. ambulation for short distances causes dyspnea and chest pain.
B. dependency on others for all transportation needs.
C. inability to safely leave home unattended.
D. leaving the home three times a week to receive dialysis.

B141 C6 1
141. A patient with an above-knee prosthesis is walking by swinging the prosthesis out to the side in an arc during
swing of the amputated limb. Your suspect the prosthesis may:
A. be too long or the knee mechanism too stiff.
B. have an excessively low lateral wall.
C. have an unstable knee unit or is too short.
D. have insufficient support from the anterior or posterior walls.

B142 C4 2
142. An 87 year-old patient has been hospitalized for three weeks following a surgical resection of carcinoma of the
colon. He is very weak and you are instructing him in ambulation with a walker. He complains of pain in his left
shoulder which is aggravated by weight bearing when using the walker. You decide to:
A. apply cold (to reduce pain) prior to ambulation training.
B. apply heat in the form of a hot pack.
C. notify his physician immediately.
D. observe the shoulder carefully over the next few sessions, documenting any changes in pain.

B143 C5 2
143. A 17 year-old high school football player sprained his left ankle two days ago. He complains of moderate pain
(5/10), there is moderate swelling which seems to be worsening; he ambulates with an antalgic gait. In this case,
the OPTIMAL interventions to choose are:
A. cold whirlpool, followed by elastic compression and elevation.
B. cold/intermittent compression combination followed by elevation.
C. contrast baths and elastic compression.
D. intermittent compression followed by elevation.

B144 C1 1
144. Your patient diagnosed with left lateral epicondylitis has no resolution of symptoms after two weeks of treatment.
You begin a re-examination and find the left biceps reflex is 1+. You should NEXT perform a complete
examination of the:
A. cervical spine.
B. shoulder.
C. thoracic spine.
D. wrist.

373406391.doc
24 Practice Questions

B145 C3 2
145. An obese patient who is 70 pounds overweight is recovering from a mild myocardial infarction and needs
cardiovascular conditioning. The exercise class will be used in conjunction with a dietary program to promote
weight reduction. The MOST appropriate exercise prescription for this patient is:
A. jogging, 10 min. mile (10.2 METs).
B. swimming, intensity set at 75% age-adjusted heart rate.
C. walking, intensity set at 50% target heart rate.
D. walking, intensity set at 85% of heart rate reserve.

B146 C1 1
146. A 68 year-old male underwent a total hip replacement (THR) four months ago. He is now referred to physical
therapy for gait evaluation. He demonstrates shortened stride length on the right. The patient MOST LIKELY has:
A. lengthened gluteals.
B. shortened hamstrings.
C. shortened hip flexors.
D. weakened quadriceps.

B147 C8 3
147. A child with spastic cerebral palsy is having difficulty releasing food from her hand into her mouth. Once the child
has brought the food to the mouth it would be helpful for the caregiver to:
A. give a quick stretch to the finger extensors.
B. passively extend the fingers.
C. slowly stroke the finger extensors in a distal to proximal direction.
D. slowly stroke the finger flexors in a distal to proximal direction.

B148 C2 2
148. A patient with amyotrophic lateral sclerosis presents with early signs of progressive muscle weakness and muscle
cramping. He also has minimal spasticity in his lower extremities. Your INITIAL exercise prescription should
focus on:
A. active exercises since resistive exercises are contraindicated.
B. low intensity general conditioning exercises.
C. maximizing exercise and activity.
D. PROM exercise to maintain joint flexibility since active exercise is contraindicated.

B149 C6 2
149. In reference to the figure, when lifting a constant load using either a stoop lift or squat lift posture, the most
significant contributing factor for increasing the lumbar spine compression forces is:
A. performing the lift with the lumbar spine in a kyphotic posture using a stoop lift technique.
B. performing the lift with the lumbar spine in a neutral position rather than a lordotic posture.
C. the distance of the load from the base of the spine.
D. the height of the load from the ground.

B150 C1 2
150. Nearly two months ago, a 50 year-old female noticed that she had left shoulder pain after walking her dog. This
pain has progressively worsened. She now is unable to move her left upper extremity overhead while performing
activities of daily living. An orthopedic surgeon diagnosed her problem as adhesive capsulitis. The MOST
effective subacute intervention for this patient would include:
A. anterior inferior translatory glides.
B. phonophoresis with dexamethasone cream.
C. posterior translatory glides.
D. use of ice packs and a sling.

373406391.doc
Practice Questions 25

B151 C8 1
151. A 79 year-old patient is referred to physical therapy for an examination of his functional mobility skills and safety
in the home environment. The family reports he is demonstrating increasing forgetfulness and some memory
deficits. In your examination you would expect to find:
A. impairments in short-term memory.
B. periods of agitation and wandering, especially in the late afternoon.
C. periods of fluctuating confusion.
D. significant impairments in long-term memory.

B152 C2 1
152. You are examining a patient with vestibular dysfunction. You have the patient sit with the head turned to one side.
You quickly move the patient backward so that the head is extended over the end of the table approximately 30
below horizontal. The maneuver causes dizziness and vertigo. You report these findings as:
A. a negative positional test.
B. a positive Clinical Test for Sensory Interaction in Balance (CTSIB).
C. a positive Hallpike maneuver.
D. a positive sharpened Romberg test.

B153 C7 3
153. You are instructing a patient with a lower extremity amputation in prosthetic gait ambulation. You determine that
learning is going well because the patients errors are decreasing and overall endurance is increasing. The BEST
strategy to promote continued motor learning is to:
A. have the patient begin to walk in the hall outside of the P.T. clinic.
B. have the patient continue to practice in the parallel bars until all errors are extinguished.
C. intervene early whenever errors appear before bad habits become firmly entrenched.
D. provide continuous feedback after every walking attempt.

B154 C3 3
154. Which intervention would NOT be appropriate for home physical therapy for a patient with cystic fibrosis?
A. activities to increase endurance.
B. breathing techniques that will decrease both respiratory rate and inspiratory volumes.
C. inspiratory muscle training to increase the strength of the ventilatory muscles.
D. secretion removal techniques to all lobes once or twice a day.

B155 C4 2
155. A 46 year-old woman is referred to your clinic with problems of stress incontinence. She reports loss of control
that began with coughing or laughing but now reports she has problems even when she exercises (she does
aerobics 3 times a week). The BEST intervention for this patient is:
A. behavioral modification techniques to reward proper voiding on schedule.
B. biofeedback one hour a week to achieve appropriate sphincter control.
C. functional electrical stimulation three times a week.
D. Kegel exercises several times a day.

B156 C2 1
156. A 76 year-old patient suffered a cerebral thrombosis four days ago. She presents with the following symptoms:
analgesia and thermoanesthesia of the ipsilateral face; nystagmus, vertigo, and nausea; dysphagia and dysarthria;
ipsilateral Horners syndrome; and contralateral loss of sensations of pain and temperature of the body. The MOST
LIKELY site of the lesion is the:
A. anterior inferior cerebellar artery.
B. basilar artery.
C. internal carotid artery.
D. posterior inferior cerebellar artery.

373406391.doc
26 Practice Questions

B157 C5 2
157. An 80 year-old patient with a two inch stage III decubitus ulcer over the left lateral malleolus is referred to you for
electrical stimulation. You note a greenish, pungent exudate at the wound site. Your choice of polarity and
electrode placement would be:
A. anode placed in the wound.
B. anode placed proximal to wound.
C. cathode placed in the wound.
D. cathode placed proximal to wound.

B158 C3 3
158. Running on level gym surface is a suitable Phase 2 outpatient cardiac rehabilitation activity for some patients
because the MET level is of:
A. high intensity (15 METs) and should only be used for late phase II training.
B. low intensity, (below 5 METs) and little variability between individuals is expected.
C. moderate intensity, (13 METs) and energy expenditure will vary significantly among individuals so it should
be used with caution.
D. moderate intensity, (8-9 METS), and can be successfully alternated with lower intensity walking, (5 METs), to
produce the right intensity.

B159 C4 2
159. A 27 year-old woman presents with a complete T10 paraplegia. An extensive neurological work-up has failed to
reveal a specific cause for her paraplegia. Her physician has determined that she has a conversion disorder. During
physical therapy it would be BEST to:
A. postpone her rehabilitation until she has had adequate psychological counseling.
B. treat this patient the same as any other patient with paraplegia.
C. try to get this patient to recognize the cause of her malingering.
D. use functional electrical stimulation as a means of demonstrating to her that the muscles function.

B160 C7 3
160. A physical therapist, a certified cardiopulmonary clinical specialist, was asked to treat a patient who had a surgical
repair of a lacerated index finger flexor tendon. This was the first hand injury that the therapist had ever treated.
During the treatment the patient felt a pop which was the result of a rupture of the newly repaired tendon. The
physical therapist in this case should have:
A. refused to treat this patient.
B. requested more time to read the literature about this repair.
C. treated the patient as requested.
D. used heat prior to the treatment.

B161 C8 1
161. Which is NOT considered a normal finding during an examination of a newborn infant?
A. continuous tremulousness.
B. dramatic skin color changes with change of state.
C. response decrement to repetitive stimuli.
D. symmetry in range of motion.

B162 C6 1
162. A patient comes into your clinic with a wheelchair with excessive leg length from the seat to the foot plate. You
suspect this is resulting in his:
A. excessive forward leaning.
B. excessive weight bearing on the ischial seat.
C. sacral sitting and sliding forward in the chair.
D. uneven thigh weight distribution.

373406391.doc
Practice Questions 27

B163 C4 1
163. A patient presents with a large sacral decubitus ulcer that is purulent and draining. You need to take a
representative sample of the infected material in order to obtain a laboratory culture. The MOST appropriate
method to culture this wound is to obtain samples from the:
A. dressing and exudate in the wound.
B. dressing, exudate, and surrounding bed linen.
C. exudate in the wound and the surrounding tissues.
D. exudate in the wound.

B164 C3 3
164. A Phase 2 outpatient cardiac rehabilitation program uses circuit training with different exercise stations for the 50
minute program. One station uses arm ergometry. For arm exercise compared to leg exercise at a given workload
you can expect:
A. both HR and systolic/diastolic BP will be higher.
B. exercise capacity is reduced due to higher stroke volumes.
C. HR will be higher while systolic BP will be lower.
D. the principal change is higher systolic and diastolic BP.

B165 C7 2
165. A therapist wants to determine whether a treatment was effective in reducing lower extremity edema in a group of
patients with peripheral vascular edema. Volumetric measurements are used using water displacement method. The
data was compared to a control group receiving no treatment. Analysis of this data is BEST done by employing:
A. ANOVA.
B. chi square.
C. Pearsons product moment.
D. t-test.

B166 C8 2
166. In planning a physical activity program for an elderly patient with Alzheimers disease, it is MOST important that
the daily activities:
A. are changed daily to meet the need for variety.
B. are highly structured to reduce anxiety and confusion.
C. promote involvement and interest through maximum stimulation.
D. provide maximum opportunities for making choices.

B167 C2 3
167. Your patient has a complete spinal cord injury at the level of C6. You are instructing his family in exercises to
maintain his passive range of motion. You want the family to focus on:
A. keeping all muscles fully ranged through normal ROM.
B. keeping muscles fully ranged, with hyperflexibility in the low back extensors and hamstrings.
C. limiting range of motion in the shoulders to promote stability.
D. ranging individual muscles according to specific functional needs.

B168 C7 3
168. A physical therapist and physical therapist assistant are conducting a cardiac rehabilitation class for twenty
patients. The therapist is suddenly called out of the room. The MOST appropriate procedure in this situation is to:
A. have the patients switch to less intense exercise until the therapist returns.
B. have the PTA supervise the class in cool-down activities.
C. have the PTA supervise the class using the outlined exercise protocol until the therapist returns.
D. terminate the exercises and have the patients monitor their pulses until the therapist returns.

B169 C1 3
169. The most efficient intervention to regain biceps brachii strength if the muscle is chronically inflamed and has a
painful arc of motion is:
A. biceps curls through partial range of motion
B. isokinetic exercises through the full range of motion.
C. isometric exercises at 10 degree intervals through the range of motion.
D. isometric exercises at the end range of movement.

373406391.doc
28 Practice Questions

B170 C5 3
170. During an ultrasound treatment your patient flinches and states that a strong ache is felt in the treatment area. To
accommodate to this patients complaint, it would be BEST to:
A. add more transmission medium.
B. decrease the ultrasound frequency.
C. decrease the ultrasound intensity.
D. increase the size of the treatment area.

B171 C7 3
171. The Back to Work Center, which specializes in work conditioning, is scheduled for an accrediting site survey. The
appropriate agency to conduct this program is the:
A. Commission on Accreditation of Rehabilitation Facilities (CARF).
B. Department of Health & Human Services
C. Joint Commission on Accreditation of Health Care Organizations (JCAHO).
D. Occupational Safety & Health Administration (OSHA).

B172 C4 2
172. You are currently treating a 32 year-old patient recovering from deep partial thickness burns over the posterior
thigh and calf which are now healed. Your examination reveals local tenderness with swelling and pain on
movement in the hip area. As you inspect the tissues you detect a palpable mass. Your BEST course of action is to:
A. cancel therapy for that day to allow tenderness to subside.
B. continue with ROM exercises but proceed gently.
C. report these findings promptly to the physician.
D. use RICE to quiet down the inflammatory response.

B173 C8 1
173. A 10 month-old (corrected age) infant born at 23 weeks gestational age suffered Grade III intraventricular
hemorrhage prenatally and was on a ventilator for 2 months after birth. During a physical therapy examination,
increased resistance to passive movement is noted in all extremities, but most markedly in the lower extremities.
The infant will probably be diagnosed with:
A. ataxic cerebral palsy.
B. spastic diplegic cerebral palsy.
C. spastic hemiplegic cerebral palsy.
D. spastic quadriplegic cerebral palsy.

B174 C3 1
174. ECG changes that occur with exercise in an individual with myocardial ischemia and coronary artery disease
would be expected to include:
A. bradycardia with ST segment depression greater than 3 mm below baseline.
B. significant arrhythmias early on in exercise with a shortened QRS.
C. significant tachycardia early on in exercise with ST segment elevation.
D. tachycardia at a relatively low intensity of exercise with ST segment depression.

B175 C1 2
175. A 45 year-old female patient with long term postural changes exhibits an excessive forward head and complains of
pain and dizziness when looking upward. The MOST effective physical therapy intervention is:
A. anterior cervical muscle stretching and postural reeducation to relieve vertebral artery compression.
B. posterior cervical muscle stretching and postural reeducation to relieve vertebral artery compression.
C. upper cervical Grade III mobilization to reduce pressure on the cervical spinal nerves.
D. upper cervical joint mobilization Grade II and postural reeducation to reduce compression of the cervical
sympathetic ganglia.

B176 C6 1
176. Your patient has an above-knee prosthesis with a prosthetic foot that includes a plantar flexion bumper. During
ambulation you decide that the bumper is too stiff. You base this decision on the presence of:
A. foot slap.
B. high heel rise.
C. lateral foot rotation at heel strike.
D. swing phase whip.

373406391.doc
Practice Questions 29

B177 C3 3
177. During the course of the physical therapy treatment in the ICU, a radial artery line gets pulled (comes out of the
artery). The FIRST thing the physical therapist should do is:
A. call for the nurse and elevate the arm above heart level.
B. place a brachial blood pressure cuff on the involved extremity and inflate the cuff to a level above systole.
C. push the code button in the patients room and elevate the arm above heart level.
D. re-insert the arterial catheter into the radial artery and inflate the pressure pack to a level above systole.

B178 C2 1
178. A 52 year-old patient suffers a CVA resulting in right hemisphere damage. This patient will MOST LIKELY
exhibit:
A. hesitancy, requiring more feedback and support.
B. negative, self-deprecating comments and frequent depression.
C. poor judgment with increased safety issues.
D. slow, cautious behaviors.

B179 C4 1
179. A 64 year-old patient is referred to your clinic with a diagnosis of herpes zoster with neuralgic pain that has been
intractable for the last 4 months. She has been given systemic corticosteroids as part of her medical treatment. It is
NOT expected that this drug intervention would result in:
A. hypokalemia with hypotension and arrhythmias.
B. increased susceptibility to infection.
C. muscle cramps, tetany and paresthesias.
D. osteoporosis and myopathy.

B180 C1 1
180. After examining a female patient who was referred to physical therapy for posterior thoracic pain, you have found
no musculoskeletal causes for the patients symptoms. Pain may be referred to the thoracic region from the:
A. appendix.
B. gall bladder.
C. heart.
D. ovary.

B181 C6 3
181. An 82 year-old patient with a transfemoral amputation is to be fitted with a temporary prosthesis containing a
SACH prosthetic foot. This prosthetic foot:
A. absorbs energy through a series of bumpers, permitting sagittal plane motion only.
B. allows full sagittal and frontal plane motion.
C. allows limited sagittal plane motion with a small amount of mediolateral motion.
D. is an articulated foot with multiplanar motion.

B182 C8 2
182. An 82 year-old patient demonstrates significant proprioceptive losses in both lower extremities, distal greater than
proximal. Effective compensatory strategies to assist in ambulation training would consist of:
A. constant verbal cuing concerning his foot position.
B. having him watch the position of his feet while standing.
C. limiting his ambulation and focusing on wheelchair mobility.
D. walking on smooth tile floors.

B183 C3 1
183. A patient is referred for inpatient cardiac rehabilitation. He has a diagnosis of sinus bradycardia. You observe the
patient carefully for worsening signs and symptoms of sinus bradycardia that include:
A. fibrillatory waves instead of P waves on his ECG.
B. hypotension, chest pain, and dizziness.
C. palpitations and racing HR.
D. ST segment depression with T wave flattening.

373406391.doc
30 Practice Questions

B184 C5 3
184. Your patient complains of pain in the shoulder region secondary to subdeltoid bursitis. Pain is reported to be 9/10.
As part of the plan of care during the acute phase, you elect to use conventional TENS which will modulate the
pain primarily by:
A. ascending inhibition.
B. descending inhibition.
C. gate control mechanisms.
D. stimulation of endorphins.

B185 C1 1
185. A thirty-five year-old male laborer describes a sudden onset of back pain while trying to lift a heavy barrel with
another worker. He describes his pain as constant, unremitting at an intensity of 10/10 over the last three days. He
is unable to work, but can drive himself to the clinic for treatment. Medication has not influenced his symptoms.
He states he has never had any other back-related symptoms in the past. MOST LIKELY involved in his
symptomatology is:
A. discal dysfunction.
B. early degenerative osteoarthritis.
C. neoplastic disease.
D. secondary gain.

B186 C3 1
186. Following a myocardial infarction, a patient was placed on medications which included a beta-adrenergic blocking
agent. When monitoring this patients response to exercise, you expect this drug will cause heart rate to:
A. be low at rest and rise linearly as a function of increasing workload.
B. be low at rest and rise very little with exercise.
C. increase proportionally to changes in diastolic blood pressure.
D. increase proportionally to changes in systolic blood pressure.

B187 C7 3
187. You are working in an outpatient clinic that has provider contracts with many types of insurance plans. A large
percentage of the clinics patients have insurance coverage that has a contract that pays a flat rate of
reimbursement per episode of care. Your supervisor tells you never to treat these patients for more than five visits.
You should:
A. ignore your supervisor and treat the patients at the frequency and duration appropriate for their condition.
B. keep track of all your patients insured by this plan, and make certain they average less than or equal to five
visits in aggregate.
C. resign from the clinic and report the supervisor to the states licensing board.
D. speak to your supervisor about concerns you have that the policy is in conflict with the Code of Ethics.

B188 C4 3
188. Your patient is a 65 year-old with a 20 year history of diabetes. Changes include vascular insufficiency and
diminished sensation of both feet with poor healing of a superficial skin lesion. It is important that he understand
the precautions and guidelines on foot care for people with diabetes. All of the following will BEST accomplish
this goal except instructing the patient to:
A. inspect the skin daily for inflammation, swelling, redness, blisters, or wounds.
B. use daily hot soaks of feet followed by an application of petroleum jelly to moisturize skin.
C. wash the feet daily with mild soap, dry thoroughly, and hydrate with moisturizing lotion.
D. wear oxford-type or jogging shoes that allow adequate room and change shoes frequently.

B189 C1 1
189. A patient complained of an inability to fully extend his knee when running. During your examination you
determine that he cannot fully extend his knee while positioned in supine with the foot dorsiflexed and the hip
flexed first to 60 degrees and then 90 degrees. The tightness is most likely caused by the:
A. gastrocnemius.
B. hamstrings.
C. psoas.
D. soleus.

373406391.doc
Practice Questions 31

B190 C1 2
190. A nineteen year-old male soccer player sustained a Grade II inversion ankle sprain two weeks ago. Interventions in
the early subacute phase of rehabilitation should include:
A. closed-chain lower extremity strengthening, proprioceptive exercises, and an orthosis.
B. lower extremity strengthening for soccer and other ADL-related tasks.
C. open-chain lower extremity exercises, crutch training, and ankle wrap.
D. rest, ice, compression and elevation (RICE).

B191 C5 2
191. A 46 year-old patient was referred to you following a tendon transfer of the extensor carpi radialis longus. The
muscle strength tested 2/5 in spite of previous intensive therapy. You choose biofeedback to assist in progressively
increasing active motor recruitment. Initially, the EMG biofeedback protocol should consist of:
A. high detection sensitivity with recording electrodes placed closely together.
B. high detection sensitivity with recording electrodes placed far apart.
C. low detection sensitivity with recording electrodes placed closely together.
D. low detection sensitivity with recording electrodes placed far apart.

B192 C8 1
192. You receive a referral from the nurse case manager for physical therapy evaluation for an ambulation program. An
82 year-old has lost functional independence lately after the recent death of his wife. His past medical history
includes stroke with minimal residual disability. Currently he no longer goes out of his house and rarely even gets
out of his chair anymore. During the initial session you determine that depression is likely the cause of his
increasing inactivity based on the presence of:
A. complaints of increasing dizziness and palpitations
B. low scores on the Geriatric Depression Scale.
C. sleep apnea
D. withdrawal, symptoms of fatigue and weight loss.

B193 C7 3
193. A 40 year-old ironworker has been laid off his construction job for the last two years. He has a wife who is a
homemaker and seven small children. They are currently on welfare. While raking leaves he has a stroke and is
admitted to the hospital. In this case, the third party payer that would provide assistance is:
A. Medicaid.
B. Medicare Part A.
C. Social Security.
D. Workers Compensation

B194 C3 1
194. A patient has been on bedrest for two days following revascularization surgery involving triple coronary artery
bypass graft. During his therapy session you suspect he may be developing complications of deep vein
thrombophlebitis. The MOST important signs of this potential medical emergency are:
A. dyspnea and tachypnea with leg pain.
B. pain in the calf after exercising, and diaphoresis.
C. paresthesias and a cyanotic cold extremity.
D. tenderness, aching, and swelling in the calf.

B195 C1 1
195. A patient referred to you for TMJ dysfunction states that she has had three episodes of her jaw locking in an open
position. The MOST LIKELY cause of her problems is the:
A. disc.
B. lateral pterygoid muscle.
C. masseter muscle
D. retrodiscal lamina.

373406391.doc
32 Practice Questions

B196 C4 2
196. A 62 year-old patient is referred to physical therapy after a fall and ORIF for a fracture of her right wrist. During
your initial examination, you observe that her skin and eyes have a yellowish hue. When you question her about
this she tells you that she noticed it herself starting about 3 or 4 days ago. Your BEST course of action is to:
A. document your findings and consult with her physician by phone after the treatment session.
B. document your findings and continue on with her treatment.
C. send a written copy of your examination, including your findings of her integumentary condition to her
referring physician.
D. write up your examination, then treat her problem with whirlpool and massage.

B197 C1 2
197. A 75 year-old inpatient received a cemented total hip replacement two days ago. The physical therapy plan of care
should have as its number one priority:
A. active range of motion exercises and early ambulation using a walker, nonweightbearing.
B. passive range of motion exercises and gait training using crutches, weightbearing to tolerance.
C. patient education regarding positions and movements to avoid.
D. use of a CPM machine followed by gait training in the parallel bars.

B198 C7 3
198. A patient is referred to you by an orthopedist with the diagnosis of impingement syndrome of the shoulder. The
initial examination reveals signs and symptoms that are not consistent with this diagnosis and more consistent with
thoracic spine pain and dysfunction. You treat the patient consistent with your findings without communicating
with the referring physician. Months later you are sued by the patients estate for failure to order a chest X-ray. The
patient died of undiagnosed metastatic lung cancer. You are:
A. not legally licensed to order a chest X-ray, therefore cannot be held responsible for the patients death from
metastatic lung cancer.
B. not responsible for the incorrect diagnosis because your treatment was appropriate for the findings identified
on the initial examination.
C. responsible for communicating findings to the referring physician when the findings are inconsistent with a
referring physicians diagnosis.
D. responsible for making this diagnosis of possible cancer consistent with your examination of this patient.

B199 C5 3
199. A 44 year-old patient with lymphedema is receiving intermittent compression to the left lower extremity. An
appropriate treatment time for home administration would be up to:
A. 2 hours/3 times per week.
B. 30 minutes/daily.
C. 8 hours/3 times per week.
D. 8 hours/daily.

B200 C6 2
200. A patient with a 10 year history of multiple sclerosis demonstrates 3+ extensor tone in both lower extremities. You
need to order her a wheelchair. It would be BEST to recommend a:
A. electric wheelchair with toe loops.
B. standard wheelchair with a 30 degree reclining back.
C. standard wheelchair with elevating leg rests.
D. tilt-in-space wheelchair with a pelvic belt.

373406391.doc
Practice Questions 33

1. reaching for a multicolored object while in an unsupported, guarded sitting position.


2. change to grade II mobilizations to reduce the joint and soft tissue irritation.
3. use frequent reinforcements for all desired behaviors.
4. isokinetic training using low resistance at fast movement speeds.
5. halo orthosis.
6. administer supplemental O2.
7. electrical silence.
8. passive ROM exercise.
9. stretching of scalenes and sternocleidomastoid muscles for early thoracic outlet syndrome symptoms.
10. hypoglycemia; you immediately administer orange juice and notify his physician.
11. modify her exercise prescription to decrease the intensity and duration but maintain a frequency of 3 times/week.
12. provide real-life examples that link learning to her personal experiences.
13. daily, following the initial examination.
14. exercise-induced bronchospasm.
15. isotonic exercises using ankle weights.
16. strengthen hip extensors through bridging.
17. Sulfamylon
18. cataracts
19. an environmental and daily structure in which the patient is best able to process stimuli cognitively.
20. sitting, weight bearing on extended left arm, weight shifting.
21. the spread of scores with cold treatment demonstrates variability is greater.
22. supraspinatus
23. restrict all movement.
24. weak deltoids.
25. the infants head is turned to the right.
26. provide a questionnaire to all participants two weeks before the scheduled session.
27. maximum use of demonstration and gesture.
28. provide suggestions to nursing for positioning for optimal motor development.
29. closed-chain functional strengthening of the quadriceps femoris and hamstrings emphasizing regaining terminal
knee extension.
30. a black and white face, with a red nose, held approximately nine inches from the infants eyes, moved horizontally.
31. position in sidelying, check to see if the airway is open, and immediately call for emergency assistance.
32. dependent variable.
33. slowed respiratory rate.
34. posterior plastic shell.
35. Stage 1 ulcer formation.
36. four electrodes with current flow diagonal to the spinal column.
37. 9-10 months.
38. postural reeducation, soft tissue mobilization, and dynamic stabilization.
39. ask about previous work and other experiences related to transferring individuals.
40. skilled nursing facility.
41. taste sensation over the anterior tongue as well as motor function of facial muscles.
42. decreased trunk rotation with shorter steps.
43. orthostatic hypotension and dizziness.
44. within a few days following surgery to preserve tendon gliding.
45. decreased pain and tenderness at the fracture sites.
46. optic chiasm.
47. common peroneal nerve.
48. accuracy of the measurements increases at higher saturation levels.
49. anterior cruciate ligament.
50. pendulum exercises.
51. intense cold, burning, and aching, followed by numbness.
52. individual facilities modifying its format and implementation schedule.
53. practice isolated small range quadriceps eccentric control work in standing and continue with the straight cane.
54. progressive resistance training for the quadriceps.
55. the patients employer when the condition is work-related.
56. combining the usage of a lumbar support, armrests, and increase the angle between the seat pan and backrest to
between 90-110 degrees.
57. suprascapular nerve.

373406391.doc
34 Practice Questions

58. needs assessment performed by a physical therapist.


59. repetition of movements and positions that provoke dizziness and vertigo.
60. inadequate knee friction or too little tension in the extension aid.
61. give him some water to drink and report your findings to his physician immediately.
62. greater than 1 mm ST segment depression, horizontal or downsloping.
63. neutral warmth.
64. ipsilateral weakness and loss of position sense and vibration below the lesion level with contralateral loss of pain
& temperature sensation.
65. demonstrate the task, then practice with the patient.
66. home health agency rehabilitation services.
67. socket is aligned too far forward or tilted anteriorly.
68. sustained maximal inspiration training with an incentive spirometer.
69. a high back.
70. phonophoresis to reduce inflammation of the tendinous sheaths.
71. a pommel to keep hips abducted.
72. sensory losses and sensory organization of balance.
73. the PT who is solely responsible for assessing the competence of the PTA under their supervision.
74. complete the written component of the discharge summary at another time and evaluate this new patient.
75. ankle plantarflexion and anterior pelvic tilt.
76. L4 nerve root compression.
77. take his BP pre and post exercise, using his non-shunt arm.
78. three-point.
79. since a no treatment group was not used for comparison, no conclusions can be reached.
80. stop the exercise and notify the physician immediately.
81. visual agnosia.
82. drugs that produce essential tremor.
83. examination for an orthosis.
84. grasp the handrims posteriorly and pull them forward abruptly and forcefully.
85. amyotrophic lateral sclerosis with dysphagia and diminished gag reflex.
86. partial squats progressing to lunges.
87. low lumbar (L4-L5).
88. paraffin with the patient positioned in a recumbent position.
89. front wheel rolling walker that folds.
90. forward, producing posterior tilting of the pelvis.
91. demonstrate PROM techniques, offering corrections as they perform the exercises for you.
92. facial nerve pathology: Bells palsy.
93. standard postural drainage for the lateral basal segment, left lower lobe.
94. these cognitive symptoms are intermittent and come on suddenly, often at night.
95. analysis of variance.
96. utilize demonstration and gesture to get the idea across of what you want him to do.
97. insufficient knee friction.
98. plastic cervical orthosis and axillary splints (airplane position).
99. compression of the common peroneal nerve.
100. bridging with arms at sides.
101. give the patients family Medicare notification of noncoverage information, and carefully explain it, and their
options, which could include paying for the care out-of-pocket.
102. call the patients physician due to signs of theophylline toxicity.
103. there may be a dislocation at C1/C2 due to ligamentous laxity.
104. passive range of motion while emphasizing scapulohumeral rhythm.
105. weakness or severe spasticity of the quadriceps.
106. povidone-iodine.
107. leg ergometry is highly correlated with RPE while arm ergometry is only moderately correlated.
108. immediately return the patient to the referring physician with documentation indicating that treatment was
ineffective.
109. continuous US @ 3 MHz.
110. facilitation of early movement in synergistic patterns followed quickly by movement patterns out-of-synergy.
111. bilateral KAFOs.
112. strengthen the hip abductors on the right side.
113. decreased loading of the knee by the quadriceps femoris muscle.

373406391.doc
Practice Questions 35

114. calcium alginate dressings.


115. centralized gnawing pain with loss of postural control during lifting activities.
116. involving the patient in decision making, emphasizing safety and independent performance.
117. use a toe wedge.
118. high intensity workloads for short durations.
119. increased HR and RR along with dilated pupils.
120. ambulation for exercise with a reciprocating gait orthosis and Lofstrand crutches.
121. placebo effect.
122. respiratory alkalosis.
123. avoid exercise during periods of peak insulin activity.
124. will likely be resistant to activity training if unfamiliar activities are used.
125. focus on knowledge of results and visual inputs.
126. pad over the transverse arch.
127. modalities to reduce pain and inflammation as the result of subdeltoid bursitis.
128. alterations in memory, confusion, and disorientation.
129. the applicants marital status.
130. cause femoral anteversion and knee strain.
131. dorsal column/lemniscal pathways or somatosensory cortex.
132. several times a week using interval training, for 10 to 15 minute periods.
133. a statement ensuring the subjects commitment to participate for the duration of the study.
134. sacroiliac joints.
135. skin and electrodes.
136. improve positioning for pressure relief or for patients with extensor spasms.
137. practice walking in areas of high illumination and low clutter.
138. angina increasing in intensity and unresponsive to the nitroglycerin or rest.
139. foot tapping in a sitting position.
140. dependency on others for all transportation needs.
141. be too long or the knee mechanism too stiff.
142. notify his physician immediately.
143. cold/intermittent compression combination followed by elevation.
144. cervical spine.
145. walking, intensity set at 50% target heart rate.
146. shortened hip flexors.
147. slowly stroke the finger extensors in a distal to proximal direction.
148. low intensity general conditioning exercises.
149. the distance of the load from the base of the spine.
150. anterior inferior translatory glides.
151. impairments in short-term memory.
152. a positive Hallpike maneuver.
153. have the patient begin to walk in the hall outside of the P.T. clinic.
154. breathing techniques that will decrease both respiratory rate and inspiratory volumes.
155. Kegel exercises several times a day.
156. posterior inferior cerebellar artery.
157. cathode placed in the wound.
158. moderate intensity, (8-9 METS), and can be successfully alternated with lower intensity walking, (5 METs), to
produce the right intensity.
159. treat this patient the same as any other patient with paraplegia.
160. refused to treat this patient.
161. continuous tremulousness.
162. sacral sitting and sliding forward in the chair.
163. exudate in the wound.
164. both HR and systolic/diastolic BP will be higher.
165. t-test.
166. are highly structured to reduce anxiety and confusion.
167. ranging individual muscles according to specific functional needs.
168. have the PTA supervise the class using the outlined exercise protocol until the therapist returns.
169. isometric exercises at 10 degree intervals through the range of motion.
170. decrease the ultrasound intensity.
171. Commission on Accreditation of Rehabilitation Facilities (CARF).

373406391.doc
36 Practice Questions

172. report these findings promptly to the physician.


173. spastic diplegic cerebral palsy.
174. tachycardia at a relatively low intensity of exercise with ST segment depression.
175. posterior cervical muscle stretching and postural reeducation to relieve vertebral artery compression.
176. lateral foot rotation at heel strike.
177. place a brachial blood pressure cuff on the involved extremity and inflate the cuff to a level above systole.
178. poor judgment with increased safety issues.
179. muscle cramps, tetany and paresthesias.
180. gall bladder.
181. allows limited sagittal plane motion with a small amount of mediolateral motion.
182. having him watch the position of his feet while standing.
183. hypotension, chest pain, and dizziness.
184. gate control mechanisms.
185. secondary gain.
186. be low at rest and rise very little with exercise.
187. speak to your supervisor about concerns you have that the policy is in conflict with the Code of Ethics.
188. use daily hot soaks of feet followed by an application of petroleum jelly to moisturize skin.
189. gastrocnemius.
190. closed-chain lower extremity strengthening, proprioceptive exercises, and an orthosis.
191. high detection sensitivity with recording electrodes placed closely together.
192. withdrawal, symptoms of fatigue and weight loss.
193. Medicaid.
194. tenderness, aching, and swelling in the calf.
195. disc.
196. document your findings and consult with her physician by phone after the treatment session.
197. patient education regarding positions and movements to avoid.
198. responsible for communicating findings to the referring physician when the findings are inconsistent with a
referring physicians diagnosis.
199. 8 hours/daily.
200. tilt-in-space wheelchair with a pelvic belt.

373406391.doc

You might also like